Transcript
Page 1: Solution manual for intermediate accounting-14e-kieso-warfield-weygandt
Page 2: Solution manual for intermediate accounting-14e-kieso-warfield-weygandt

TABLE OF CONTENTS

CHAPTER 1 ............................................................................................................ 1

Concepts for Analysis 1-5 .................................................................................. 1 Concepts for Analysis 1-6 .................................................................................. 2 Concepts for Analysis 1-10 ................................................................................ 3 Concepts for Analysis 1-11 ................................................................................ 3

CHAPTER 2 ............................................................................................................ 4

Brief Exercise 2-3 .............................................................................................. 4 Brief Exercise 2-4 .............................................................................................. 4 Brief Exercise 2-5 .............................................................................................. 4 Exercise 2-3 ....................................................................................................... 5

CHAPTER 3 ............................................................................................................ 6

Exercise 3-6 ....................................................................................................... 6 Exercise 3-9 ....................................................................................................... 6 Exercise 3-11 ..................................................................................................... 8 Exercise 3-14 ................................................................................................... 10 Exercise 3-15 ................................................................................................... 10 Exercise 3-16 ................................................................................................... 10

CHAPTER 4 .......................................................................................................... 12

Exercise 4-2 ..................................................................................................... 12 Exercise 4-4 ..................................................................................................... 13 Exercise 4-5 ..................................................................................................... 16 Exercise 4-12 ................................................................................................... 17 Exercise 4-13 ................................................................................................... 18 Exercise 4-15 ................................................................................................... 19 Problem 4-1 ..................................................................................................... 19 Problem 4-7 ..................................................................................................... 21

CHAPTER 5 .......................................................................................................... 23

Exercise 5-2 ..................................................................................................... 23 Exercise 5-4 ..................................................................................................... 23 Exercise 5-13 ................................................................................................... 25 Exercise 5-15 ................................................................................................... 25 Problem 5-2 ..................................................................................................... 27

CHAPTER 7 .......................................................................................................... 29

Exercise 7-5 ..................................................................................................... 29 Exercise 7-7 ..................................................................................................... 30 Exercise 7-13 ................................................................................................... 30 Exercise 7-15 ................................................................................................... 31

Page 3: Solution manual for intermediate accounting-14e-kieso-warfield-weygandt

Exercise 7-16 ................................................................................................... 31 Exercise 7-24 ................................................................................................... 32 Problem 7-8 ..................................................................................................... 34 Problem 7-11................................................................................................... 35 Problem 7-15................................................................................................... 36

CHAPTER 8 .......................................................................................................... 38

Exercise 8-1 ..................................................................................................... 38 Exercise 8-15 ................................................................................................... 38 Exercise 8-25 ................................................................................................... 39 Exercise 8-26 ................................................................................................... 40

CHAPTER 9 .......................................................................................................... 42

Brief Exercise 9-2 ............................................................................................ 42 Brief Exercise 9-4 ............................................................................................ 42 Brief Exercise 9-7 ............................................................................................ 42 Brief Exercise 9-8 ............................................................................................ 43 Exercise 9-2 ..................................................................................................... 43 Exercise 9-7 ..................................................................................................... 44 Exercise 9-12 ................................................................................................... 45 Exercise 9-14 ................................................................................................... 46 Exercise 9-19 ................................................................................................... 47 Problem 9-4 ..................................................................................................... 47

CHAPTER 18 ........................................................................................................ 49

Exercise 18-2 ................................................................................................... 49 Exercise 18-4 ................................................................................................... 50 Exercise 18-7 ................................................................................................... 51 Exercise 18-11 ................................................................................................. 52 Exercise 18-15 ................................................................................................. 53 Exercise 18-19 ................................................................................................. 54 Problem 18-7................................................................................................... 55 Problem 18-8................................................................................................... 57

CHAPTER 23 ........................................................................................................ 59

Exercise 23-1 ................................................................................................... 59 Exercise 23-5 ................................................................................................... 59 Exercise 23-6 ................................................................................................... 60 Exercise 23-11 ................................................................................................. 60

Page 4: Solution manual for intermediate accounting-14e-kieso-warfield-weygandt

1

CHAPTER 1

CA 1-5 (a) One of the committees that the AICPA established prior to the establishment of the FASB was the

Committee on Accounting Procedures (CAP). The CAP, during its existence from 1939 to 1959, issued 51 Accounting Research Bulletins (ARB). In 1959, the AICPA created the Accounting Prin-ciples Board (APB) to replace the CAP. Before being replaced by the FASB, the APB released 31 official pronouncements, called APB Opinions.

(b) Although the ARBs issued by the CAP helped to narrow the range of alternative practices to some

extent, the CAP’s problem-by-problem approach failed to provide the well-defined, structured body of accounting principles that was both needed and desired. As a result, the CAP was replaced by the APB.

The APB had more authority and responsibility than did the CAP. Unfortunately, the APB was beleaguered throughout its 14-year existence. It came under fire early, charged with lack of produc-tivity and failing to act promptly to correct alleged accounting abuses. The APB also met a lot of industry and CPA firm opposition and occasional governmental interference when tackling numerous thorny accounting issues. In fear of governmental rule making, the accounting profession investigated the ineffectiveness of the APB and replaced it with the FASB.

Learning from prior experiences, the FASB has several significant differences from the APB. The FASB has: (1) smaller membership, (2) full-time, compensated membership, (3) greater autonomy, (4) increased independence, and (5) broader representation. In addition, the FASB has its own research staff and relies on the expertise of various task force groups formed for various projects. These features form the bases for the expectations of success and support from the public. In addition, the due process taken by the FASB in establishing financial accounting standards gives interested persons ample opportunity to make their views known. Thus, the FASB is responsive to the needs and viewpoints of the entire economic community, not just the public accounting profession.

(c) The AICPA has supplemented the FASB’s efforts in the present standard-setting environment. The issue papers, which are prepared by the Accounting Standards Executive Committee (AcSEC), identify current financial reporting problems for specific industries and present alternative treat-ments of the issue. These papers provide the FASB with an early warning device to insure timely issuance of FASB standards, Interpretations, and Staff Positions. In situations where the FASB avoids the subject of an issue paper, AcSEC may issue a Statement of Position to provide guidance for the reporting issue. AcSEC also issues Practice Bulletins which indicate how the AICPA believes a given transaction should be reported.

Recently, the role of the AICPA in standard-setting has diminished. The FASB and the AICPA agreed, that after a transition period, the AICPA and AcSEC no longer will issue authoritative accounting guidance for public companies.

CA 1-6

Page 5: Solution manual for intermediate accounting-14e-kieso-warfield-weygandt

2

(a) The Financial Accounting Foundation (FAF) is the sponsoring organization of the FASB. The FAF selects the members of the FASB and its Advisory Council, funds their activities, and generally oversees the FASB’s activities.

The FASB follows a due process in establishing a typical FASB Statement of Financial Accounting Standards. The following steps are usually taken: (1) A topic or project is identified and placed on the Board’s agenda. (2) A task force of experts from various sectors is assembled to define problems, issues, and alternatives related to the topic. (3) Research and analysis are conducted by the FASB technical staff. (4) A preliminary views document is drafted and released. (5) A public hearing is often held, usually 60 days after the release of the preliminary views. (6) The Board analyzes and evaluates the public response. (7) The Board deliberates on the issues and prepares an exposure draft for release. (8) After a 30-day (minimum) exposure period for public comment, the Board evaluates all of the responses received. (9) A committee studies the exposure draft in relation to the public responses, reevaluates its position, and revises the draft if necessary. (10) The full Board gives the revised draft final consideration and votes on issuance of a Standards Statement. The passage of a new accounting standard in the form of an FASB Statement requires the support of five of the seven Board members.

(b) The FASB issues three major types of pronouncements: Standards and Interpretations, Financial

Accounting Concepts, and Technical Bulletins. Financial accounting standards issued by the FASB are considered GAAP. In addition, the FASB also issues interpretations that represent modifications or extensions of existing standards and APB Opinions. These interpretations have the same authority as standards and APB Opinions in guiding current accounting practices.

The Statements of Financial Accounting Concepts (SFAC) help the FASB to avoid the “problem-by-problem approach.” These statements set forth fundamental objectives and concepts that the Board will use in developing future standards of financial accounting and reporting. They are intended to form a cohesive set of interrelated concepts, a body of theory or a conceptual framework, that will serve as tools for solving existing and emerging problems in a consistent, sound manner.

The FASB may issue a technical bulletin when there is a need for guidelines on implementing or applying FASB Standards or Interpretations, APB Opinions, Accounting Research Bulletins, or emerging issues. A technical bulletin is issued only when (1) it is not expected to cause a major change in accounting practice for a number of enterprises, (2) its cost of implementation is low, and (3) the guidance provided by the bulletin does not conflict with any broad fundamental accounting principle. In addition, the FASB’s Emerging Issues Task Force (EITF) issues statements to provide guidance on how to account for new and unusual financial transactions that have the potential for creating diversity in reporting practices. The EITF identifies controversial accounting problems as they arise and determines whether they can be quickly resolved or whether the FASB should become involved in solving them. In essence, it becomes a “problem filter” for the FASB. Thus, it is hoped that the FASB will be able to work on more pervasive long-term problems, while the EITF deals with short-term emerging issues.

CA 1-10 1. (b), (e) 2. (a) 3. (c)

Page 6: Solution manual for intermediate accounting-14e-kieso-warfield-weygandt

3

4. (d)

CA 1-11 1. (d) 2. (f) 3. (c) 4. (e) 5. (a) 6. (b)

CHAPTER 2

BRIEF EXERCISE 2-3

(a) Equity

(b) Revenues

(c) Equity

(d) Assets

(e) Expenses

(f) Losses

(g) Liabilities

(h) Distributions to owners

(i) Gains

(j) Investments by owners

BRIEF EXERCISE 2-4

(a) Periodicity

(b) Monetary unit

(c) Going concern

(d) Economic entity

BRIEF EXERCISE 2-5

(a) Revenue recognition

Page 7: Solution manual for intermediate accounting-14e-kieso-warfield-weygandt

4

(b) Expense recognition

(c) Full disclosure

(d) Historical cost

EXERCISE 2-3 (15–20 minutes)

(a) Gains, losses.

(b) Liabilities.

(c) Investments by owners, comprehensive income.

(also possible would be revenues and gains).

(d) Distributions to owners.

(Note to instructor: net effect is to reduce equity and assets).

(e) Comprehensive income.

(also possible would be revenues and gains).

(f) Assets.

(g) Comprehensive income.

(h) Revenues, expenses.

(i) Equity.

(j) Revenues.

(k) Distributions to owners.

(l) Comprehensive income.

Page 8: Solution manual for intermediate accounting-14e-kieso-warfield-weygandt

5

CHAPTER 3

EXERCISE 3-6 (10–15 minutes)

1. Accounts Receivable .............................................................................................................. 750

Service Revenue ........................................................................................................... 750

2. Utilities Expense ..................................................................................................................... 520

Utilities Payable ............................................................................................................ 520

3. Depreciation Expense ............................................................................................................ 400

Accumulated Depreciation—Dental Equipment .......................................................... 400

Interest Expense .................................................................................................................... 500

Interest Payable ............................................................................................................ 500

4. Insurance Expense ($15,000 X 1/12) ..................................................................................... 1,250

Prepaid Insurance ......................................................................................................... 1,250

5. Supplies Expense ($1,600 – $400) ......................................................................................... 1,200

Supplies ........................................................................................................................ 1,200

EXERCISE 3-9 (15–20 minutes)

(a) 10/15 Salaries Expense .................................................................................................................... 800

Cash ............................................................................................................................. 800

Page 9: Solution manual for intermediate accounting-14e-kieso-warfield-weygandt

6

(To record payment of October 15

payroll)

10/17 Accounts Receivable ............................................................................................................. 2,100

Service Revenue .......................................................................................................... 2,100

(To record revenue for services

performed for which payment has

not yet been received)

10/20 Cash 650

Unearned Service Revenue ......................................................................................... 650

(To record receipt of cash for

services not yet performed)

(b) 10/31 Supplies Expense ................................................................................................................... 470

Supplies ........................................................................................................................ 470

(To record the use of supplies during

October)

10/31 Accounts Receivable ............................................................................................................. 1,650

Service Revenue .......................................................................................................... 1,650

(To record revenue for services

performed for which payment

has not yet been received)

10/31 Salaries Expense .................................................................................................................... 600

Salaries Payable ........................................................................................................... 600

(To record liability for accrued payroll)

10/31 Unearned Service Revenue ................................................................................................... 400

Service Revenue .......................................................................................................... 400

(To reduce the Unearned Service

Revenue account for service that

Page 10: Solution manual for intermediate accounting-14e-kieso-warfield-weygandt

7

has been performed)

EXERCISE 3-11 (20–25 Minutes)

(a) CAVAMANLIS CO.

Income Statement

For the Year Ended December 31, 2010

Revenues

Service revenue ....................................................................... $12,590

Expenses

Salaries expense ...................................................................... $6,840

Rent expense ........................................................................... 2,760

Depreciation expense .............................................................. 145

Interest expense ...................................................................... 83 9,828

Net Income ........................................................................................... $ 2,762

(b) CAVAMANLIS CO.

Statement of Retained Earnings

For the Year Ended December 31, 2010

Retained earnings, January 1 ........................................................................................ $11,310

Add: Net income .................................................................................................... 2,762

Less: Dividends ............................................................................................................. 3,000

Retained earnings, December 31 ................................................................................. $11,072

(c) CAVAMANLIS CO.

Balance Sheet

Page 11: Solution manual for intermediate accounting-14e-kieso-warfield-weygandt

8

December 31, 2010

Assets

Current Assets

Cash $18,972

Accounts receivable ............................................................. 6,920

Prepaid rent ......................................................................... 2,280

Total current assets ..................................................... $28,172

Property, plant, and equipment

Equipment ............................................................................ 18,050

Less: Accumulated depreciation ......................................... (4,895) 13,155

Total assets $41,327

Liabilities and Stockholders’ Equity

Current liabilities

Notes payable ...................................................................... $ 5,700

Accounts payable ................................................................. 4,472

Interest payable ................................................................... 83

Total current liabilities ................................................. 10,255

Stockholders’ equity

Common Stock ..................................................................... $20,000

Retained Earnings ................................................................ 11,072* 31,072

Total liabilities and stockholders’ equity .............................................. $41,327

*Beg. Balance + Net Income – Dividends = Ending Balance

$11,310 + $2,762 – $3,000 = $11,072

EXERCISE 3-14 (10–15 minutes)

Sales 340,000

Sales Returns and Allowances ..................................................................................... 13,000

Page 12: Solution manual for intermediate accounting-14e-kieso-warfield-weygandt

9

Sales Discounts ............................................................................................................ 8,000

Income Summary ......................................................................................................... 319,000

Income Summary ................................................................................................................... 302,000

Cost of Goods Sold ....................................................................................................... 202,000

Freight-out ................................................................................................................... 7,000

Insurance Expense ....................................................................................................... 12,000

Rent Expense ............................................................................................................... 20,000

Salary Expense ............................................................................................................. 61,000

Income Summary ................................................................................................................... 17,000

Retained Earnings ........................................................................................................ 17,000

EXERCISE 3-15 (10–15 minutes)

(a) $5,000 ($90,000 – $85,000) (d) $95,000 ($5,000 + $90,000)

(b) $29,000 ($85,000 – $56,000) (e) $52,000 ($90,000 – $38,000)

(c) $14,000 ($29,000 – $15,000)

EXERCISE 3-16 (10–15 minutes)

Sales 390,000

Cost of Goods Sold ....................................................................................................... 235,700

Sales Returns and Allowances ..................................................................................... 12,000

Sales Discounts ............................................................................................................ 15,000

Selling Expenses ........................................................................................................... 16,000

Administrative Expenses .............................................................................................. 38,000

Income Tax Expense .................................................................................................... 30,000

Income Summary ......................................................................................................... 43,300

(or)

Page 13: Solution manual for intermediate accounting-14e-kieso-warfield-weygandt

10

Sales 390,000

Income Summary ......................................................................................................... 390,000

Income Summary ................................................................................................................... 346,700

Cost of Goods Sold ....................................................................................................... 235,700

Sales Returns and Allowances ..................................................................................... 12,000

Sales Discounts ............................................................................................................ 15,000

Selling Expenses ........................................................................................................... 16,000

Administrative Expenses .............................................................................................. 38,000

Income Tax Expense .................................................................................................... 30,000

Income Summary ................................................................................................................... 43,300

Retained Earnings ........................................................................................................ 43,300

Retained Earnings .................................................................................................................. 18,000

Dividends ..................................................................................................................... 18,000

CHAPTER 4

EXERCISE 4-2 (25–35 minutes)

(a) Total net revenue:

Page 14: Solution manual for intermediate accounting-14e-kieso-warfield-weygandt

11

Sales ............................................................... $400,000

Less: Sales discounts ..................................... $ 7,800

Sales returns .................................... 12,400 20,200

Net sales ......................................................... 379,800

Dividend revenue ........................................... 71,000

Rental revenue ............................................... 6,500

Total net revenue ............................... $457,300

(b) Net income:

Total net revenue (from a)............................. $457,300

Expenses:

Cost of goods sold .................................... $184,400

Selling expenses........................................ 99,400

Administrative expenses .......................... 82,500

Interest expense ....................................... 12,700

Total expenses .................................... 379,000

Income before income tax ............................. 78,300

Income tax ..................................................... 26,600

Net income ............................................... $ 51,700

(c) Dividends declared:

Ending retained earnings ............................... $134,000

Beginning retained earnings .......................... 114,400

Net increase ............................................................... 19,600

Less: Net income (from (b)) ...................................... 51,700

Dividends declared .................................................... $ 32,100

ALTERNATE SOLUTION (for (c))

Page 15: Solution manual for intermediate accounting-14e-kieso-warfield-weygandt

12

Beginning retained earnings .......................................................... $114,400

Add: Net income ...................................................................... 51,700

166,100

Less: Dividends declared ............................................................... ?

Ending retained earnings ............................................................... $134,000

Dividends declared must be $32,100

($166,100 – $134,000)

EXERCISE 4-4 (30–35 minutes)

(a) Multiple-Step Form

WEBSTER COMPANY

Income Statement

For the Year Ended December 31, 2010

(In thousands, except earnings per share)

Sales $96,500

Cost of goods sold ....................................................................... 63,570

Gross profit .................................................................................. 32,930

Operating Expenses

Selling expenses

Sales commissions ............................................................ $7,980

Depr. of sales equipment ................................................. 6,480

Transportation-out ........................................................... 2,690 $17,150

Administrative expenses

Officers’ salaries ............................................................... 4,900

Depr. of office furn. and equip. ........................................ 3,960 8,860 26,010

Income from operations ............................................. 6,920

Page 16: Solution manual for intermediate accounting-14e-kieso-warfield-weygandt

13

Other Revenues and Gains

Rental revenue ....................................................................... 17,230

24,150

Other Expenses and Losses

Interest expense .................................................................... 1,860

Income before income tax .......................................................... 22,290

Income tax .............................................................................. 7,580

Net income .................................................................................. $14,710

Earnings per share ($14,710 ÷ 40,550) ....................................... $.36

(b) Single-Step Form

WEBSTER COMPANY

Income Statement

For the Year Ended December 31, 2010

(In thousands, except earnings per share)

Revenues

Sales $ 96,500

Rental revenue .................................................................................................. 17,230

Total revenues ............................................................................................. 113,730

Expenses

Cost of goods sold ............................................................................................. 63,570

Selling expenses ................................................................................................ 17,150

Administrative expenses ................................................................................... 8,860

Interest expense ............................................................................................... 1,860

Total expenses ............................................................................................. 91,440

Income before income tax ..................................................................................... 22,290

Income tax .............................................................................................................. 7,580

Page 17: Solution manual for intermediate accounting-14e-kieso-warfield-weygandt

14

Net income........................................................................................................ $ 14,710

Earnings per share .................................................................................................. $0.36

Note: An alternative income statement format for the single-step form is to show income tax as

part of expenses, and not as a separate item.

(c) Single-step:

1. Simplicity and conciseness.

2. Probably better understood by users.

3. Emphasis on total costs and expenses and net income.

4. Does not imply priority of one revenue or expense over another.

Multiple-step:

1. Provides more information through segregation of operating and nonoperating items.

2. Expenses are matched with related revenue.

Note to instructor: Students’ answers will vary due to the nature of the question; i.e., it asks for an

opinion. However, the discussion supporting the answer should include the above points.

EXERCISE 4-5 (30–35 minutes)

PARNEVIK CORP.

Income Statement

For the Year Ended December 31, 2010

Sales Revenue

Page 18: Solution manual for intermediate accounting-14e-kieso-warfield-weygandt

15

Sales $1,280,000

Less: Sales returns and allowances ........................................................ $150,000

Sales discounts ....................................................................... 45,000 195,000

Net sales revenue .................................................................................... 1,085,000

Cost of goods sold ................................................................................... 621,000

Gross profit ................................................................................................... 464,000

Operating Expenses

Selling expenses................................................................................ 194,000

Admin. and general expenses .......................................................... 97,000 291,000

Income from operations ............................................................................... 173,000

Other Revenues and Gains

Interest revenue .................................................................................. 86,000

259,000

Other Expenses and Losses

Interest expense .................................................................................. 60,000

Income before tax and extraordinary item ................................................... 199,000

Income tax ($199,000 X .34) ............................................................... 67,660

Income before extraordinary item ................................................................ 131,340

Extraordinary item—loss from earthquake damage ........................................ 120,000

Less: Applicable tax reduction ($120,000 X .34) .................................... 40,800 79,200

Net income .................................................................................................... $ 52,140

Per share of common stock:

Income before extraordinary item

($131,340 ÷ 100,000) ..........................................................................

$1.31*

Extraordinary item (net of tax) .............................................................. (0.79)

Net income ($52,140 ÷ 100,000)............................................................ $0.52

Page 19: Solution manual for intermediate accounting-14e-kieso-warfield-weygandt

16

*Rounded

EXERCISE 4-12 (15–20 minutes)

Net income:

Income from continuing operations

before income tax ........................................................................................................

$21,650,000

Income tax (35% X $21,650,000) ..................................................................................... 7,577,500

Income from continuing operations ................................................................................ 14,072,500

Discontinued operations

Loss before income tax ............................................................................................... $3,225,000

Less: Applicable income tax (35%) ............................................................................ 1,128,750 2,096,250

Net income ....................................................................................................................... $11,976,250

Preferred dividends declared: ............................................................................................... $ 860,000

Weighted average common shares outstanding ................................................................................. 4,000,000

Earnings per share

Income from continuing operations ................................................................................ $3.30*

Discontinued operations, net of tax ................................................................................ (0.52)**

Net income ....................................................................................................................... $2.78***

*($14,072,500 – $860,000) ÷ 4,000,000. (Rounded)

**$2,096,250 ÷ 4,000,000. (Rounded)

***($11,976,250 – $860,000) ÷ 4,000,000.

EXERCISE 4-13 (15–20 minutes)

(a) 2010

Page 20: Solution manual for intermediate accounting-14e-kieso-warfield-weygandt

17

Income before income tax ........................................................ $460,000

Income tax (35%) ....................................................................... 161,000

Net Income ................................................................................ $299,000

(b) Cumulative effect for years prior to 2010:

Year

Weighted

Average FIFO Difference

Tax Rate

(35%) Net Effect

2008 $370,000 $395,000 $25,000

2009 390,000 420,000 30,000

Total $55,000 $19,250 $35,750

(c) 2010 2009 2008

Income before income tax $460,000 $420,000 $395,000

Income tax (35%) 161,000 147,000 138,250

Net income $299,000 $273,000 $256,750

EXERCISE 4-15 (15–20 minutes)

BRYANT CO.

Statement of Stockholders’ Equity

For the Year Ended December 31, 2010

Compre-

hensive

Retained

Accumulated

Other

Comprehensive

Common

Page 21: Solution manual for intermediate accounting-14e-kieso-warfield-weygandt

18

Total Income Earnings Income Stock

Beginning balance $520,000 $ 90,000 $80,000 $350,000

Comprehensive income

Net income* 170,000 $170,000 170,000

Other comprehensive income

Unrealized holding loss (50,000) (50,000) (50,000)

Comprehensive income $120,000

Dividends (10,000) (10,000)

Ending balance $630,000 $250,000 $30,000 $350,000

*($750,000 – $500,000 – $80,000).

SOLUTIONS TO PROBLEMS

PROBLEM 4-1

DICKINSON COMPANY

Income Statement

For the Year Ended December 31, 2010

Sales ............................................................................................................... $25,000,000

Cost of goods sold ......................................................................................... 16,000,000

Gross profit .................................................................................................... 9,000,000

Selling and administrative expenses ............................................................. 4,700,000

Income from operations ................................................................................ 4,300,000

Other revenues and gains

Interest revenue .............................................................................. $ 70,000

Gain on the sale of investments ...................................................... 110,000 180,000

Other expenses and losses

Write-off of goodwill ....................................................................... 820,000

Income from continuing operations before income tax ......................................... 3,660,000

Income tax ..................................................................................................... 1,244,000

Income from continuing operations ............................................................. 2,416,000

Page 22: Solution manual for intermediate accounting-14e-kieso-warfield-weygandt

19

Discontinued operations

Loss on operations, net of tax ......................................................... 90,000

Loss on disposal, net of tax .............................................................. 440,000 530,000

Income before extraordinary item ................................................................ 1,886,000

Extraordinary item—loss from flood

damage, net of tax .....................................................................................

390,000

Net income .................................................................................................... $ 1,496,000

Earnings per share:

Income from continuing operations ................................................ $ 4.67a

Discontinued operations

Loss on operations, net of tax .............................................. $(0.18)

Loss on disposal, net of tax .................................................. (0.88) (1.06)

Income before extraordinary item ................................................... 3.61b

Extraordinary loss, net of tax ........................................................... (0.78)

Net income ....................................................................................... $ 2.83c

DICKINSON COMPANY

Retained Earnings Statement

For the Year Ended December 31, 2010

Retained earnings, January 1 ............................................................ $ 980,000

Add: Net income ......................................................................... 1,496,000

2,476,000

Less: Dividends

Preferred stock .................................................................... $ 80,000

Common stock ..................................................................... 250,000 330,000

Retained earnings, December 31 ...................................................... $ 2,146,000

a$2,416,000 – $80,000 = $4.67

Page 23: Solution manual for intermediate accounting-14e-kieso-warfield-weygandt

20

500,000 shares

b$1,886,000 – $80,000 = $3.61

500,000 shares

c$1,496,000 – $80,000 = $2.83

500,000 shares

PROBLEM 4-7

WADE CORP.

Income Statement (Partial)

For the Year Ended December 31, 2010

Income from continuing operations

before income tax ...........................................................

$1,200,000*

Income tax .............................................................. 456,000**

Income from continuing operations .................................. 744,000

Discontinued operations

Loss from operations of

discontinued subsidiary ......................................

$ 90,000

Less: Applicable income tax

reduction .......................................

34,200

$ 55,800

Loss from disposal of subsidiary ..................................... 100,000

Less: Applicable income tax

reduction ........................................

38,000

62,000

117,800

Income before extraordinary item ..................................... 626,200

Extraordinary item:

Gain on condemnation ........................................... 125,000

Less: Applicable income tax ................................... 50,000 75,000

Page 24: Solution manual for intermediate accounting-14e-kieso-warfield-weygandt

21

Net income ......................................................................... $ 701,200

Per share of common stock:

Income from continuing operations .................................................................. $4.96

Discontinued operations, net of tax ................................................................... (0.79)

Income before extraordinary item ..................................................................... 4.17

Extraordinary item, net of tax ............................................................................ 0.50

Net income ($701,200 ÷ 150,000) ...................................................................... $4.67

*Computation of income from continuing operations

before income tax:

As previously stated $1,210,000

Loss on sale of equipment [$40,000 – ($80,000 – $30,000)] (10,000)

Restated $1,200,000

**Computation of income tax expense:

$1,200,000 X .38 = $456,000

Note: The error related to the intangible asset was correctly charged to retained earnings.

Page 25: Solution manual for intermediate accounting-14e-kieso-warfield-weygandt

22

CHAPTER 5

EXERCISE 5-2 (15–20 minutes)

1. h. 11. b.

2. d. 12. f.

3. f. 13. a.

4. f. 14. h.

5 c. 15. c.

6. a. 16. b.

7. f. 17. a.

8. g. 18. a.

9. a. 19. g.

10. a. 20. f.

EXERCISE 5-4 (30–35 minutes)

GULISTAN INC.

Balance Sheet

December 31, 2010

Assets

Current assets

Cash $XXX

Less: Cash restricted for plant

expansion .....................................................

XXX

$XXX

Accounts receivable .............................................................. XXX

Less: Allowance for doubtful

accounts .......................................................

XXX

XXX

Notes receivable ................................................................... XXX

Receivables—officers ............................................................ XXX

Inventories

Finished goods ................................................................. XXX

Work in process ............................................................... XXX

Page 26: Solution manual for intermediate accounting-14e-kieso-warfield-weygandt

23

Raw materials .................................................................. XXX XXX

Total current assets .................................................... $XXX

Long-term investments

Preferred stock investments ................................................. XXX

Land held for future plant site .............................................. XXX

Cash restricted for plant expansion ...................................... XXX

Total long-term investments ...................................... XXX

Property, plant, and equipment

Buildings ................................................................................ XXX

Less: Accum. depreciation—

buildings .......................................................

XXX

XXX

Intangible assets

Copyrights ............................................................................. XXX

Total assets ..................................................................... $XXX

Liabilities and Stockholders’ Equity

Current liabilities

Accrued salaries payable ................................................................ $XXX

Notes payable, short-term ............................................................. XXX

Unearned subscriptions revenue ................................................... XXX

Unearned rent revenue ................................................................. XXX

Total current liabilities .............................................................. $XXX

Long-term debt

Bonds payable, due in four years ................................................... $XXX

Less: Discount on bonds payable ................................................... (XXX) XXX

Total liabilities ........................................................................... XXX

Stockholders’ equity

Capital stock:

Common stock .......................................................................... XXX

Page 27: Solution manual for intermediate accounting-14e-kieso-warfield-weygandt

24

Additional paid-in capital:

Paid in capital in excess of

par—common stock ...............................................................

XXX

Total paid-in capital ............................................................. XXX

Retained earnings .......................................................................... XXX

Total paid-in capital and

retained earnings..............................................................

XXX

Less: Treasury stock, at cost .................................................... (XXX)

Total stockholders’ equity ................................................... XXX

Total liabilities and stock-

holders’ equity ..................................................................

$XXX

Note to instructor: An assumption made here is that cash included the cash restricted for plant

expansion. If it did not, then a subtraction from cash would not be necessary or the cash balance would

be “grossed up” and then the cash restricted for plant expansion deducted.

EXERCISE 5-13 (15–20 minutes)

(a) 4. (f) 1. (k) 1.

(b) 3. (g) 5. (l) 2.

(c) 4. (h) 4. (m) 2.

(d) 3. (i) 5.

(e) 1. (j) 4.

EXERCISE 5-15 (25–35 minutes)

(a) SONDERGAARD CORPORATION

Statement of Cash Flows

For the Year Ended December 31, 2010

Cash flows from operating activities

Net income ........................................................................................... $160,000

Adjustments to reconcile net income

to net cash provided by operating

Page 28: Solution manual for intermediate accounting-14e-kieso-warfield-weygandt

25

activities:

Depreciation expense ..................................................................... $17,000

Loss on sale of investments ............................................................ 7,000

Decrease in accounts receivable .................................................... 5,000

Decrease in current liabilities ......................................................... (17,000) 12,000

Net cash provided by operating activities ........................................... 172,000

Cash flows from investing activities

Sale of investments

[($74,000 – $52,000) – $7,000] ......................................................... 15,000

Purchase of equipment ........................................................................ (58,000)

Net cash used by investing activities ................................................... (43,000)

Cash flows from financing activities

Payment of cash dividends .................................................................. (50,000)

Net increase in cash ................................................................................... 79,000

Cash at beginning of year .......................................................................... 78,000

Cash at end of year .................................................................................... $157,000

(b) Free Cash Flow Analysis

Net cash provided by operating activities ................................................. $172,000

Less: Purchase of equipment.................................................................... (58,000)

Dividends .................................................................................... (50,000)

Free cash flow .......................................................................................... $ 64,000

PROBLEM 5-2

Page 29: Solution manual for intermediate accounting-14e-kieso-warfield-weygandt

26

MONTOYA, INC.

Balance Sheet

December 31, 2010

Assets

Current assets

Cash $ 360,000

Trading securities ....................................................... 121,000

Notes receivable ........................................................ 445,700

Income taxes receivable ............................................ 97,630

Inventories ................................................................. 239,800

Prepaid expenses ....................................................... 87,920

Total current assets .............................................. $1,352,050

Property, plant, and equipment

Land $ 480,000

Building ...................................................................... $1,640,000

Less: Accum. depreciation—

building ..............................................

270,200

1,369,800

Equipment .................................................................. 1,470,000

Less: Accum. depreciation—

equipment .........................................

292,000

1,178,000

3,027,800

Intangible assets

Goodwill ..................................................................... 125,000

Total assets ........................................................... $4,504,850

Liabilities and Stockholders’ Equity

Page 30: Solution manual for intermediate accounting-14e-kieso-warfield-weygandt

27

Current liabilities

Accounts payable ......................................................... $ 490,000

Notes payable to banks ................................................ 265,000

Payroll taxes payable ................................................... 177,591

Taxes payable ............................................................... 98,362

Rent payable ................................................................ 45,000

Total current liabilities ............................................ $1,075,953

Long-term liabilities

Unsecured notes payable

(long-term) ............................................................... $1,600,000

Bonds payable .............................................................. $300,000

Less: Discount on bonds

payable .................................................

15,000

285,000

Long-term rental obligations ...................................... 480,000 2,365,000

Total liabilities ......................................................... 3,440,953

Stockholders’ equity

Capital stock

Preferred stock, $10 par; 20,000

shares authorized, 15,000

shares issued ........................................................

$150,000

Common stock, $1 par;

400,000 shares authorized,

200,000 issued .....................................................

200,000

$350,000

Retained earnings

($1,063,897 – $350,000) ...........................................

713,897

Total stockholders’ equity

($4,504,850 – $3,440,953) ...................................

1,063,897

Total liabilities and

stockholders’ equity .............................................

$4,504,850

CHAPTER 7

Page 31: Solution manual for intermediate accounting-14e-kieso-warfield-weygandt

28

EXERCISE 7-5 (15–20 minutes)

(a) 1. June 3 Accounts Receivable—Arquette ................................................................... 2,000

Sales ................................................................................................... 2,000

June 12 Cash ............................................................................................................... 1,960

Sales Discounts ($2,000 X 2%) ....................................................................... 40

Accounts Receivable—Arquette ....................................................... 2,000

2. June 3 Accounts Receivable—Arquette ................................................................... 1,960

Sales ($2,000 X 98%).......................................................................... 1,960

June 12 Cash ............................................................................................................... 1,960

Accounts Receivable—Arquette ....................................................... 1,960

(b) July 29 Cash .............................................................................................................. 2,000

Accounts Receivable—Arquette ........................................................... 1,960

Sales Discounts Forfeited ...................................................................... 40

(Note to instructor: Sales discounts forfeited could have been recog-nized at the time the

discount period lapsed. The company, however, would probably not record this forfeiture

until final cash settlement.)

EXERCISE 7-7 (10–15 minutes)

Page 32: Solution manual for intermediate accounting-14e-kieso-warfield-weygandt

29

(a) Bad Debt Expense .......................................................................................... 7,500

Allowance for Doubtful Accounts ...................................................... 7,500*

*.01 X ($800,000 – $50,000) = $7,500

(b) Bad Debt Expense .......................................................................................... 6,000

Allowance for Doubtful Accounts ...................................................... 6,000*

*Step 1: .05 X $160,000 = $8,000 (desired credit balance in Allowance account)

Step 2: $8,000 – $2,000 = $6,000 (required credit entry to bring allowance account to

$8,000 credit balance)

EXERCISE 7-13 (10–15 minutes)

(a) Cash................................................................................................................ 290,000

Finance Charge .............................................................................................. 10,000*

Notes Payable .................................................................................... 300,000

*2% X $500,000 = $10,000

(b) Cash................................................................................................................ 350,000

Accounts Receivable .......................................................................... 350,000

EXERCISE 7-13 (Continued)

(c) Notes Payable ................................................................................................ 300,000

Interest Expense ............................................................................................ 7,500*

Cash.................................................................................................... 307,500

*10% X $300,000 X 3/12 = $7,500

EXERCISE 7-15 (10–15 minutes)

Page 33: Solution manual for intermediate accounting-14e-kieso-warfield-weygandt

30

Computation of net proceeds:

Cash received ................................................................................................ $190,000

Less: Recourse liability ................................................................................. 2,000

Net proceeds ................................................................................................ $188,000

Computation of gain or loss:

Carrying value ............................................................................. $200,000

Net proceeds .............................................................................. 188,000

Loss on sale of receivables ......................................................... $ 12,000

The following journal entry would be made:

Cash ................................................................................................... $190,000

Loss on Sale of Receivables ............................................................... 12,000

Recourse Liability ....................................................................... 2,000

Accounts Receivable .................................................................. 200,000

EXERCISE 7-16 (15–20 minutes)

(a) To be recorded as a sale, all of the following conditions would be met:

1. The transferred asset has been isolated from the transferor (put beyond reach of the

transferor and its creditors).

2. The transferees have obtained the right to pledge or to exchange either the transferred

assets or beneficial interests in the trans-ferred assets.

3. The transferor does not maintain effective control over the trans-ferred assets through an

agreement to repurchase or redeem them before their maturity.

Page 34: Solution manual for intermediate accounting-14e-kieso-warfield-weygandt

31

(b) Computation of net proceeds:

Cash received ($250,000 X 94%) .......................................... $235,000

Due from factor ($250,000 X 4%) ......................................... 10,000 $245,000

Less: Recourse obligation .................................................... 3,000

Net proceeds ........................................................................ $242,000

Computation of gain or loss:

Carrying value .................................................................... $250,000

Net proceeds ..................................................................... 242,000

Loss on sale of receivables ................................................ $ 8,000

The following journal entry would be made:

Cash ................................................................................................... $235,000

Due from Factor ................................................................................ 10,000

Loss on Sale of Receivables ............................................................... 8,000

Recourse Liability ....................................................................... 3,000

Accounts Receivable .................................................................. 250,000

*EXERCISE 7-24 (15–20 minutes)

(a) KIPLING COMPANY

Bank Reconciliation

July 31

Balance per bank statement, July 31 ............................................................ $ 8,650

Add: Deposits in transit ................................................................................ 2,850a

Deduct: Outstanding checks ......................................................................... (1,100)b

Correct cash balance, July 31 ........................................................................ $10,400

Balance per books, July 31 ............................................................................ $ 9,250

Add: Collection of note ................................................................................ 1,500

Page 35: Solution manual for intermediate accounting-14e-kieso-warfield-weygandt

32

Less: Bank service charge ............................................................................. $ 15

NSF check ............................................................................................ 335 (350)

Corrected cash balance, July 31 .................................................................... $10,400

aComputation of deposits in transit

Deposits per books $5,810

Deposits per bank in July $ 4,500

Less deposits in transit (June) (1,540)

Deposits mailed and received

in July

(2,960)

Deposits in transit, July 31 $2,850

bComputation of outstanding checks

Checks written per books $3,100

Checks cleared by bank in July $ 4,000

Less outstanding checks

(June)*

(2,000)

Checks written and cleared

in July

(2,000)

Outstanding checks, July 31 $1,100

*Assumed to clear bank in July

(b) Cash ............................................................................................................... 1,150

Office Expenses—Bank Charges .................................................................... 15

Accounts Receivable ...................................................................................... 335

Notes Receivable ............................................................................... 1,500

PROBLEM 7-8

Page 36: Solution manual for intermediate accounting-14e-kieso-warfield-weygandt

33

10/1/10 Notes Receivable ........................................................................................... 120,000

Sales ................................................................................................... 120,000

12/31/10 Interest Receivable ........................................................................................ 2,400*

Interest Revenue ............................................................................... 2,400

*$120,000 X .08 X 3/12 = $2,400

10/1/11 Cash ............................................................................................................... 9,600*

Interest Receivable ............................................................................ 2,400

Interest Revenue ...................................................... 7,200**

*$120,000 X .08 = $9,600

**$120,000 X .08 X 9/12 = $7,200

12/31/11 Interest Receivable ........................................................................................ 2,400

Interest Revenue ................................................... 2,400

10/1/12 Cash ............................................................................................................... 9,600

Interest Receivable ............................................................................ 2,400

Interest Revenue ................................................. 7,200

Cash ............................................................................................................... 120,000

Notes Receivable ............................................................................... 120,000

Note: Entries at 10/1/11 and 10/1/12 assumes reversing entries were not made on January 1, 2011 and

January 1, 2012.

PROBLEM 7-11

Page 37: Solution manual for intermediate accounting-14e-kieso-warfield-weygandt

34

SANDBURG COMPANY

Income Statement Effects

For the Year Ended December 31, 2010

Expenses resulting from accounts receivable

assigned (Schedule 1) ................................................................................... $22,320

Loss resulting from accounts receivable

sold ($300,000 – $270,000).......................................................................... 30,000

Total expenses ........................................................................................... $52,320

Schedule 1

Computation of Expense

for Accounts Receivable Assigned

Assignment expense:

Accounts receivable assigned .................................................................... $400,000

X 80%

Advance by Keller Finance Company......................................................... 320,000

X 3% $ 9,600

Interest expense ............................................................................................. 12,720

Total expenses ........................................................................................... $22,320

Page 38: Solution manual for intermediate accounting-14e-kieso-warfield-weygandt

35

*PROBLEM 7-15

(a) The entries for the issuance of the note on January 1, 2010:

The present value of the note is: $1,200,000 X .68058 = $816,700 (Rounded by $4).

Botosan Company (Debtor):

Cash .................................................................................................... 816,700

Discount on Notes Payable ................................................................ 383,300

Note Payable............................................................................... 1,200,000

National Organization Bank (Creditor):

Notes Receivable................................................................................ 1,200,000

Discount on Notes Receivable .................................................... 383,300

Cash ............................................................................................ 816,700

(b) The amortization schedule for this note is:

SCHEDULE FOR INTEREST AND DISCOUNT AMORTIZATION—

EFFECTIVE-INTEREST METHOD

$1,200,000 Note Issued to Yield 8%

Date

Cash

Paid

Interest

Expense

Discount

Amortized

Carrying Amount

of Note

1/1/10 $ 816,700

12/31/10 $0 $ 65,336* $ 65,336 882,036**

12/31/11 0 70,563 70,563 952,599

12/31/12 0 76,208 76,208 1,028,807

12/31/13 0 82,305 82,305 1,111,112

12/31/14 0 88,888 88,888 1,200,000

Total $0 $383,300 $383,300

*$816,700 X 8% = $65,336.

**$816,700 + $65,336 = $882,036.

Page 39: Solution manual for intermediate accounting-14e-kieso-warfield-weygandt

36

(c) The note can be considered to be impaired only when it is probable that, based on current

information and events, National Organization Bank will be unable to collect all amounts due

(both principal and interest) according to the contractual terms of the loan.

(d) The loss is computed as follows:

Carrying amount of loan (12/31/11) ............................................................. $952,599a

Less: Present value of $800,000 due

in 3 years at 8% ...............................................................................

(635,064)b

Loss due to impairment ................................................................................. $317,535

aSee amortization schedule from answer (b) on page 7-66.

b$800,000 X .79383 = $635,064.

December 31, 2011

National Organization Bank (Creditor):

Bad Debt Expense ...................................................................... 317,535

Allowance for Doubtful Accounts ...................................... 317,535

Note: Botosan Company (Debtor) has no entry.

CHAPTER 8

EXERCISE 8-1 (15–20 minutes)

Items 2, 3, 5, 8, 10, 13, 14, 16, and 17 would be reported as inventory in the financial statements.

The following items would not be reported as inventory:

1. Cost of goods sold in the income statement.

4. Not reported in the financial statements.

Page 40: Solution manual for intermediate accounting-14e-kieso-warfield-weygandt

37

6. Cost of goods sold in the income statement.

7. Cost of goods sold in the income statement.

9. Interest expense in the income statement.

11. Advertising expense in the income statement.

12. Office supplies in the current assets section of the balance sheet.

15. Not reported in the financial statements.

18. Short-term investments in the current asset section of the balance sheet.

EXERCISE 8-15 (15–20 minutes)

(a) ESPLANADE COMPANY

Computation of Inventory for Product

BAP Under FIFO Inventory Method

March 31, 2010

Units Unit Cost Total Cost

March 26, 2010 ................................................. 600 $12.00 $ 7,200

February 16, 2010............................................. 800 11.00 8,800

January 25, 2010 (portion) ............................... 100 10.00 1,000

March 31, 2010, inventory ............................... 1,500 $17,000

(b) ESPLANADE COMPANY

Computation of Inventory for Product

BAP Under LIFO Inventory Method

March 31, 2010

Units Unit Cost Total Cost

Beginning inventory ......................................... 600 $8.00 $ 4,800

January 5, 2010 (portion) ................................. 900 9.00 8,100

March 31, 2010, inventory ............................... 1,500 $12,900

(c) ESPLANADE COMPANY

Computation of Inventory for Product

BAP Under Weighted Average Inventory Method

March 31, 2010

Units Unit Cost Total Cost

Beginning inventory ......................................... 600 $ 8.00 $ 4,800

January 5, 2010 ................................................. 1,100 9.00 9,900

Page 41: Solution manual for intermediate accounting-14e-kieso-warfield-weygandt

38

January 25, 2010 ............................................... 1,300 10.00 13,000

February 16, 2010............................................. 800 11.00 8,800

March 26, 2010 ................................................. 600 12.00 7,200

4,400 $43,700

Weighted average cost

($43,700 ÷ 4,400) ........................................... $ 9.93*

March 31, 2010, inventory ............................... 1,500 $ 9.93 $14,895

*Rounded off.

EXERCISE 8-25 (20–25 minutes)

Current $

Price Index

Base Year $

Change from Prior

Year

2007 $ 80,000 1.00 $ 80,000 —

2008 111,300 1.05 106,000 +$26,000

2009 108,000 1.20 90,000 (16,000)

2010 122,200 1.30 94,000 +4,000

2011 147,000 1.40 105,000 +11,000

2012 176,900 1.45 122,000 +17,000

Page 42: Solution manual for intermediate accounting-14e-kieso-warfield-weygandt

39

Ending Inventory—Dollar-value LIFO:

2007 $80,000 2011 $80,000 @ 1.00 = $ 80,000

10,000 @ 1.05 = 10,500

2008 $80,000 @ 1.00 = $ 80,000 4,000 @ 1.30 = 5,200

26,000 @ 1.05 = 27,300 11,000 @ 1.40 = 15,400

$107,300 $111,100

2009 $80,000 @ 1.00 = $ 80,000 2012 $80,000 @ 1.00 = $ 80,000

10,000 @ 1.05 = 10,500 10,000 @ 1.05 = 10,500

$ 90,500 4,000 @ 1.30 = 5,200

11,000 @ 1.40 = 15,400

2010 $80,000 @ 1.00 = $ 80,000 17,000 @ 1.45 = 24,650

10,000 @ 1.05 = 10,500 $135,750

4,000 @ 1.30 = 5,200

$ 95,700

EXERCISE 8-26 (15–20 minutes)

Date

Current $

Price Index

Base-Year $

Change from

Prior Year

Dec. 31, 2007 $ 70,000 1.00 $70,000 —

Dec. 31, 2008 88,200 1.05 84,000 +$14,000

Dec. 31, 2009 95,120 1.16 82,000 (2,000)

Dec. 31, 2010 108,000 1.20 90,000 +8,000

Dec. 31, 2011 100,000 1.25 80,000 (10,000)

Page 43: Solution manual for intermediate accounting-14e-kieso-warfield-weygandt

40

Ending Inventory—Dollar-value LIFO:

Dec. 31, 2007 $70,000

Dec. 31, 2008 $70,000 @ 1.00 = $70,000

14,000 @ 1.05 = 14,700

$84,700

Dec. 31, 2009 $70,000 @ 1.00 = $70,000

12,000 @ 1.05 = 12,600

$82,600

Dec. 31, 2010 $70,000 @ 1.00 = $70,000

12,000 @ 1.05 = 12,600

8,000 @ 1.20 = 9,600

$92,200

Dec. 31, 2011 $70,000 @ 1.00 = $70,000

10,000 @ 1.05 = 10,500

$80,500

Page 44: Solution manual for intermediate accounting-14e-kieso-warfield-weygandt

41

CHAPTER 9

BRIEF EXERCISE 9-2

Item

Cost

Designated

Market

LCM

Jokers $2,000 $2,050 $2,000

Penguins 5,000 4,950 4,950

Riddlers 4,400 4,550 4,400

Scarecrows 3,200 3,070 3,070

BRIEF EXERCISE 9-4

Group

Number

of CDs

Sales

Price

per CD

Total

Sales

Price

Relative

Sales

Price

Total

Cost

Cost

Allocated to

CDs

Cost per

CD

1 100 $ 5 $ 500 5/100* X $8,000 = $ 400 $ 4**

2 800 $10 8,000 80/100 X $8,000 = 6,400 $ 8

3 100 $15 1,500 15/100 X $8,000 = 1,200 $12

$10,000 $8,000

*$500/$10,000 = 5/100 **$400/100 = $4

BRIEF EXERCISE 9-7

Beginning inventory ................................................................................ $150,000

Purchases ................................................................................................. 500,000

Cost of goods available ............................................................................ 650,000

Sales $700,000

Less gross profit (35% X 700,000)............................................................ 245,000

Estimated cost of goods sold ................................................................... 455,000

Estimated ending inventory destroyed in fire ......................................... $195,000

Page 45: Solution manual for intermediate accounting-14e-kieso-warfield-weygandt

42

BRIEF EXERCISE 9-8

Cost Retail

Beginning inventory ....................................................................... $ 12,000 $ 20,000

Net purchases ................................................................................. 120,000 170,000

Net markups ................................................................................... 10,000

Totals $132,000 200,000

Deduct:

Net markdowns .............................................................................. 7,000

Sales 147,000

Ending inventory at retail ............................................................... $ 46,000

Cost-to-retail ratio: $132,000 ÷ $200,000 = 66%

Ending inventory at lower-of cost-or-market (66% X $46,000) = $30,360

EXERCISE 9-2 (10–15 minutes)

Item

Net Realizable

Value (Ceiling)

Net Realizable

Value Less

Normal Profit

(Floor)

Replacement Cost

Designated

Market

Cost

LCM

D $90* $70** $120 $90 $75 $75

E 80 60 72 72 80 72

F 60 40 70 60 80 60

G 55 35 30 35 80 35

H 80 60 70 70 50 50

I 60 40 30 40 36 36

*Estimated selling price – Estimated selling expense = $120 – $30 = $90. **Net realizable value – Normal profit margin = $90 – $20 = $70.

Page 46: Solution manual for intermediate accounting-14e-kieso-warfield-weygandt

43

EXERCISE 9-7 (15–20 minutes)

Co

st

Pe

r Lo

t

(Co

st

Allo

cate

d/

No

. of

Lots

)

$2

,04

0

2,7

20

1,3

60

Co

st

Allo

cate

d

to L

ots

$1

8,3

60

40

,80

0

25

,84

0

$8

5,0

00

Tota

l C

ost

$8

5,0

00

85

,00

0

85

,00

0

X

X

X

Rel

ativ

e Sa

les

Pri

ce

$2

7,0

00

/$1

25,0

00

$6

0,0

00

/$1

25,0

00

$3

8,0

00

/$1

25,0

00

$7

8,0

00

53

,04

0

24

,96

0

18

,20

0

$ 6

,76

0

G

ross

P

rofi

t

$ 3

,84

0

10

,24

0

10

,88

0

$2

4,9

60

Tota

l Sa

les

Pri

ce

$

27

,00

0

6

0,0

00

3

8,0

00

$1

25

,00

0

Sa

les

(see

sch

edu

le)

Co

st o

f go

od

s so

ld (

see

sch

edu

le)

Gro

ss p

rofi

t

Op

erat

ing

exp

ense

s

Net

inco

me

Sale

s

$1

2,0

00

32

,00

0

34

,00

0

$7

8,0

00

Sale

s P

rice

Per

Lo

t

$3

,00

0

4,0

00

2,0

00

Co

st

C

ost

of

Pe

r

Lo

ts

Lo

t

So

ld

$2

,04

0

$

8,1

60

2,7

20

21

,76

0

1,3

60

23

,12

0

$

53

,04

0

No

. o

f Lo

ts

9

15

19

N

um

ber

o

f Lo

ts S

old

*

4

8

17

29 * 9

– 5

= 4

1

5 –

7 =

8

19

– 2

=

17

Page 47: Solution manual for intermediate accounting-14e-kieso-warfield-weygandt

44

Gro

up

1

Gro

up

2

Gro

up

3

Gro

up

1

Gro

up

2

Gro

up

3

To

tal

EXERCISE 9-12 (10–15 minutes)

(a) Inventory, May 1 (at cost) ................................................................ $160,000

Purchases (at cost) ........................................................................... 640,000

Purchase discounts .......................................................................... (12,000)

Freight-in .......................................................................................... 30,000

Goods available (at cost) ...................................................... 818,000

Sales (at selling price) ....................................................................... $1,000,000

Sales returns (at selling price) .......................................................... (70,000)

Net sales (at selling price) ................................................................ 930,000

Less: Gross profit (25% of $930,000) .............................................. 232,500

Sales (at cost) ....................................................................... 697,500

Approximate inventory,

May 31 (at cost) ...............................................................

$120,500

(b) Gross profit as a percent of sales must be computed:

25% = 20% of sales.

100% + 25%

Inventory, May 1 (at cost) .......................................................... $160,000

Purchases (at cost) ..................................................................... 640,000

Purchase discounts .................................................................... (12,000)

Freight-in .................................................................................... 30,000

Goods available (at cost) ................................................ 818,000

Sales (at selling price) ................................................................. $1,000,000

Sales returns (at selling price) .................................................... (70,000)

Net sales (at selling price) .......................................................... 930,000

Less: Gross profit (20% of $930,000) ........................................ 186,000

Page 48: Solution manual for intermediate accounting-14e-kieso-warfield-weygandt

45

Sales (at cost) ................................................................. 744,000

Approximate inventory,

May 31 (at cost) .........................................................

$ 74,000

EXERCISE 9-14

Beginning inventory ............................................................................... $170,000

Purchases ................................................................................................ 450,000

620,000

Purchase returns .................................................................................... (30,000)

Goods available (at cost) ........................................................................ 590,000

Sales $650,000

Sales returns ........................................................................................... (24,000)

Net sales ................................................................................................. 626,000

Less: Gross profit (30% X $626,000) ...................................................... (187,800) 438,200

Estimated ending inventory (unadjusted for

damage) ...............................................................................................

151,800

Less: Goods on hand—undamaged (at cost)

$21,000 X (1 – 30%) .................................................................

(14,700)

Less: Goods on hand—damaged (at net

realizable value) .......................................................................

(5,300)

Fire loss on inventory ............................................................................. $131,800

EXERCISE 9-19 (12–17 minutes)

Cost Retail

Beginning inventory .............................................. $ 200,000 $ 280,000

Page 49: Solution manual for intermediate accounting-14e-kieso-warfield-weygandt

46

Purchases ............................................................... 1,425,000 2,140,000

Totals ......................................................... 1,625,000 2,420,000

Add: Net markups

Markups .................................................... $95,000

Markup cancellations ............................... _________ (15,000) 80,000

Totals $1,625,000 2,500,000

Deduct: Net markdowns

Markdowns ................................................ 35,000

Markdown cancellations ........................... (5,000) 30,000

Sales price of goods available ................................ 2,470,000

Deduct: Sales ........................................................ 2,250,000

Ending inventory at retail ...................................... $ 220,000

Cost-to-retail ratio = $1,625,000

= 65% $2,500,000

Ending inventory at cost = 65% X $220,000 = $143,000

PROBLEM 9-4

Beginning inventory .................................................................................. $ 80,000

Purchases ................................................................................................... 290,000

370,000

Purchase returns ....................................................................................... (28,000)

Page 50: Solution manual for intermediate accounting-14e-kieso-warfield-weygandt

47

Total goods available ................................................................................. 342,000

Sales $415,000

Sales returns .............................................................................................. (21,000)

394,000

Less: Gross profit (35% of $394,000) ........................................................ 137,900 (256,100)

Ending inventory (unadjusted for damage) .............................................. 85,900

Less: Goods on hand—undamaged

($30,000 X [1 – 35%]) ................................................................. 19,500

Inventory damaged ................................................................................... 66,400

Less: Salvage value of damaged inventory ............................................... 8,150

Fire loss on inventory ................................................................................ $ 58,250

CHAPTER 18

EXERCISE 18-2 (15–20 minutes)

(a) 1. 6/3 Accounts Receivable—Ann Mount .......................... 8,000

Sales ............................................................. 8,000

6/5 Sales Returns and Allowances ............................................ 600

Accounts Receivable—Ann Mount ........................... 600

Page 51: Solution manual for intermediate accounting-14e-kieso-warfield-weygandt

48

6/7 Transportation-Out ............................................................. 24

Cash ........................................................................... 24

6/12 Cash ......................................................................... 7,252

Sales Discounts (2% X $7,400) ............................................ 148

Accounts Receivable—Ann Mount ........................... 7,400

2. 6/3 Accounts Receivable—Ann Mount ..................................... 7,840

Sales [$8,000 – (2% X $8,000)] ................................. 7,840

6/5 Sales Returns and Allowances ............................................ 588

Accounts Receivable—Ann Mount

[$600 – (2% x $600)] ........................................... 588

6/7 Transportation-Out ............................................................. 24

Cash ........................................................................... 24

6/12 Cash ......................................................................... 7,252

Accounts Receivable—Ann Mount ........................... 7,252

(b) 8/5 Cash ......................................................................... 7,400

Accounts Receivable—Ann Mount ........................... 7,252

Sales Discounts Forfeited

(2% X $7,400) ......................................................... 148

Page 52: Solution manual for intermediate accounting-14e-kieso-warfield-weygandt

49

EXERCISE 18-4 (20–25 minutes)

(a) Gross profit recognized in:

2010 2011 2012

Contract price $1,600,000 $1,600,000 $1,600,000

Costs: Costs to date $400,000 $825,000 $1,070,000 Estimated costs to complete

600,000

1,000,000

275,000

1,100,000

0

1,070,000

Total estimated profit

600,000

500,000

530,000

Percentage completed to date

40%*

75%**

100%

Total gross profit recognized

240,000

375,000

530,000

Less: Gross profit recognized in previous years

0

240,000

375,000

Gross profit recognized in current year

$ 240,000

$ 135,000

$ 155,000

**$400,000 ÷ $1,000,000**$825,000 ÷ $1,100,000

(b) Construction in Process ($825,000 – $400,000) ................................ 425,000

Materials, Cash, Payables, etc. ........................................................... 425,000

Accounts Receivable ($900,000 – $300,000) ..................................... 600,000

Billings on Construction in Process .......................................... 600,000

Cash ($810,000 – $270,000) ............................................................... 540,000

Accounts Receivable ................................................................. 540,000

Construction Expenses ....................................................................... 425,000

Construction in Process ...................................................................... 135,000

Revenue from Long-Term Contracts ........................................ 560,000*

*$1,600,000 X (75% – 40%)

Page 53: Solution manual for intermediate accounting-14e-kieso-warfield-weygandt

50

(c) Gross profit recognized in:

2010 2011 2012

Gross profit $–0– $–0– $530,000*

*$1,600,000 – $1,070,000

EXERCISE 18-7 (25–30 minutes)

(a) 1. Gross profit recognized in 2010:

Contract price ....................................................................... $1,200,000

Costs:

Costs to date ............................................................... $280,000

Estimated additional costs ......................................... 520,000 800,000

Total estimated profit........................................................... 400,000

Percentage completion to date

($280,000/$800,000) ........................................................ 35%

Gross profit recognized in 2010 ........................................... $ 140,000

Gross profit recognized in 2011:

Contract price ....................................................................... $1,200,000

Costs:

Costs to date ............................................................... $600,000

Estimated additional costs ......................................... 200,000 800,000

Total estimated profit........................................................... 400,000

Percentage completion to date

($600,000/$800,000) ........................................................ 75%

Total gross profit recognized ................................................ 300,000

Less: Gross profit recognized in 2010 .................................. 140,000

Gross profit recognized in 2011 ........................................... $ 160,000

2. Construction in Process ($600,000 – $280,000) ................................ 320,000

Materials, Cash, Payables, etc. ............................................... 320,000

Accounts Receivable ($500,000 – $150,000) ................................... 350,000

Billings on Construction in Process ........................................ 350,000

Cash ($320,000 – $120,000) ............................................................. 200,000

Accounts Receivable ............................................................... 200,000

Page 54: Solution manual for intermediate accounting-14e-kieso-warfield-weygandt

51

Construction in Process .................................................................... 160,000

Construction Expenses ..................................................................... 320,000

Revenues from Long-Term Contracts..................................... 480,000*

*$1,200,000 X [($600,000 – $280,000) ÷ $800,000]

(b) Income Statement (2011)—

Gross profit on long-term construction contract ......................................... $160,000

Balance Sheet (12/31/11)—

Current assets:

Receivables—construction in process ................................................ $180,000*

Inventories—construction in process totaling

$900,000** less billings of $500,000 .............................................. $400,000

**$180,000 = $500,000 – $320,000

**Total cost to date $600,000

2010 Gross profit 140,000

2011 Gross profit 160,000

$900,000

EXERCISE 18-11 (15–20 minutes)

(a) Computation of gross profit recognized:

2010 2011

$370,000 X 34%* $125,800

$350,000 X 34%* $119,000

$450,000 X 32%** 144,000

$125,800 $263,000

*($900,000 – $594,000) ÷ $900,000

**($1,000,000 – $680,000) ÷ $1,000,000(b)Installment Accounts Receivable—2011

1,000,000

Installment Sales ................................................................. 1,000,000

Cost of Installment Sales ............................................................... 680,000

Inventory ............................................................................. 680,000

Page 55: Solution manual for intermediate accounting-14e-kieso-warfield-weygandt

52

Cash 800,000

Installment Accounts Receivable, 2010 ....................................... 350,000

Installment Accounts Receivable, 2011 ....................................... 450,000 Installment Sales ................................................................................ 1,000,000

Cost of Installment Sales .......................................................... 680,000

Deferred Gross Profit on Installment

Sales, 2011 ............................................................................ 320,000 Deferred Gross Profit on Installment Sales, 2010 ................................... 119,000

Deferred Gross Profit on Installment Sales, 2011 ................................... 144,000

Realized Gross Profit on Installment Sales ................................ 263,000 Realized Gross Profit on Installment

Sales ................................................................................................ 263,000

Income Summary ...................................................................... 263,000

EXERCISE 18-15 (10–15 minutes)

(a) Realized gross profit recognized in 2011 under the installment-sales method of accounting is

$83,000. When gross profit is expressed as a percentage of cost, it must be converted to percentage of

sales to compute the realized gross profit under the installment-sales method of accounting. Thus, 2010

and 2011 gross profits as a percentage of sales are 20% and 21.875% respectively.

Sale Year

Gross Profit Percentage

2011

Collections

2011

Realized Profit

2010 .25/(1.00 + .25) = 20% $240,000 $48,000

2011 .28/(1.00 + .28) = 21.875% 160,000 35,000

TOTAL $83,000

(Note to instructor: The problem provides gross profit as a percent of cost.)

(b) The balance of “Deferred Gross Profit” could be reported on the balance sheet for 2011:

1. As a current liability on the theory that it is related to Installment Accounts Receivables that

are normally treated as current assets;

Page 56: Solution manual for intermediate accounting-14e-kieso-warfield-weygandt

53

2. As a deferred credit between liabilities and stockholders’ equity. This treatment is criticized

because there is no obligation to outsiders; or

3. As an adjustment or offset to the related Installment Accounts Receivable. This is because the

deferred gross profit is a part of revenue from installment sales not yet realized. The related

receivable will be overstated unless the deferred gross profit is deducted.

On the other hand, the amount of deferred gross profit has no direct relationship with the

estimated collectability of the accounts receivable.

It is not a settled matter as to the proper classification of “deferred gross profit” on the balance

sheet when the installment-sales method of accounting is used to measure income. As indicated

in the text, the FASB in Statement of Financial Accounting Concepts No. 6 indicates that it

conceptually is an asset valuation. We support the FASB position.

(c) Gross profit as a percent of sales in 2010 is 20% (as computed in (a) above); gross profit therefore

is $96,000 ($480,000 X .20) and the cost of 2010 sales is $384,000 ($480,000 – $96,000). Because

the amounts collected in 2010 ($130,000) and 2011 ($240,000) do not exceed the total cost of

$384,000, no profit is recognized in 2010 or 2011 on 2010 sales. Also, no profit is recognized on

2011 sales since the collections of $160,000 do not exceed the total cost of $484,375 [$620,000 X (1 –

.21875)].

*EXERCISE 18-19 (14–18 minutes)

(a) Cash ........................................................................................... 28,000

Notes Receivable .................................................................................... 42,000

Discount on Notes Receivable

[$42,000 – (2.48685 X $14,000)] .............................................. 7,184

Revenue from Franchise Fees ...................................................... 62,816

(b) Cash ........................................................................................... 28,000

Unearned Franchise Fees ............................................................. 28,000

(c) Cash ........................................................................................... 28,000

Notes Receivable .................................................................................... 42,000

Discount on Notes Receivable ...................................................... 7,184

Revenue from Franchise Fees ...................................................... 28,000

Unearned Franchise Fees

($14,000 X 2.48685) .................................................................. 34,816

(Calculations rounded)

Page 57: Solution manual for intermediate accounting-14e-kieso-warfield-weygandt

54

PROBLEM 18-7

(a) Computation of Recognizable Profit/Loss

Percentage-of-Completion Method

2010

Costs to date (12/31/10) ............................................................................ $ 300,000

Estimated costs to complete ...................................................................... 1,200,000

Estimated total costs ........................................................................ $1,500,000

Percent complete ($300,000 ÷ $1,500,000) ............................................... 20%

Revenue recognized ($1,900,000 X 20%) ................................................... $ 380,000

Costs incurred ............................................................................................. 300,000

Profit recognized in 2010 ........................................................................... $ 80,000

2011

Costs to date (12/31/11) ............................................................................ $1,200,000

Estimated costs to complete ...................................................................... 800,000

Estimated total costs ........................................................................ 2,000,000

Contract price ............................................................................................. 1,900,000

Total loss ..................................................................................................... $ 100,000

Total loss ..................................................................................................... $ 100,000

Plus gross profit recognized in 2010 .......................................................... 80,000

Loss recognized in 2011 ............................................................................. $ 180,000

OR

Percent complete ($1,200,000 ÷ $2,000,000) ............................................ 60%

Revenue recognized in 2011

[($1,900,000 X 60%) – $380,000] ........................................................... $ 760,000

Costs incurred in 2011

($1,200,000 – $300,000) ........................................................................ 900,000

Loss to date ................................................................................................ 140,000

Page 58: Solution manual for intermediate accounting-14e-kieso-warfield-weygandt

55

Loss attributable to 2012* ......................................................................... 40,000

Loss recognized in 2011 ............................................................................. $ 180,000

*2012 revenue

($1,900,000 – $380,000 – $760,000) ....................... $760,000

2012 estimated costs ........................................ 800,000

2012 loss ............................................................ $ (40,000)

2012

Costs to date (12/31/12) ................................................................. $2,100,000

Estimated costs to complete ........................................................... 0

2,100,000

Contract price .................................................................................. 1,900,000

Total loss .......................................................................................... $ (200,000)

Total loss .......................................................................................... $ (200,000)

Less: Loss recognized in 2011 ......................................................... $180,000

Gross profit recognized in 2010 ...................................................... (80,000) (100,000)

Loss recognized in 2012 .................................................................. $ (100,000)

(b) Computation of Recognizable Profit/Loss

Completed-Contract Method

2010—NONE

2011

Costs to date (12/31/11) .................................................................... $1,200,000

Estimated costs to complete ............................................................. 800,000

Estimated total costs ............................................................... 2,000,000

Deduct contract price ........................................................................ 1,900,000

Loss recognized in 2011 ..................................................................... $ (100,000)

2012

Total costs incurred ........................................................................... $2,100,000

Total revenue recognized .................................................................. 1,900,000

Page 59: Solution manual for intermediate accounting-14e-kieso-warfield-weygandt

56

Total loss on contract .............................................................. (200,000)

Deduct loss recognized in 2011 ......................................................... (100,000)

Loss recognized in 2012 ..................................................................... $ (100,000)

PROBLEM 18-8

(a)

Rate of gross profit ( Gross profit

Sales )

2010 2011 2012

38% 37% 35%

Gross profit realized:

38% of $ 75,000 $28,500

38% of $100,000 $38,000

37% of $100,000 37,000

38% of $ 50,000 $19,000

37% of $120,000 44,400

35% of $100,000 35,000

$28,500 $75,000 $98,400

(b) Installment Accounts Receivable—2012 ................................................ 280,000

Installment Sales .......................................................................... 280,000

Cash ......................................................................................... 270,000

Installment Accounts Receivable—2010 ...................................... 50,000

Installment Accounts Receivable—2011 ...................................... 120,000

Installment Accounts Receivable—2012 ...................................... 100,000

Cost of Installment Sales ........................................................................ 182,000

Inventory ...................................................................................... 182,000

Installment Sales .................................................................................... 280,000

Cost of Installment Sales .............................................................. 182,000

Deferred Gross Profit on Installment

Sales—2012 .............................................................................. 98,000

Deferred Gross Profit on Installment Sales—2010 ................................... 19,000

Deferred Gross Profit on Installment Sales—2011 ................................... 44,400

Deferred Gross Profit on Installment Sales—2012 ................................... 35,000

Page 60: Solution manual for intermediate accounting-14e-kieso-warfield-weygandt

57

Realized Gross Profit on Installment

Sales ......................................................................................... 98,400

Realized Gross Profit on Installment Sales ............................................ 98,400

Income Summary ......................................................................... 98,400

CHAPTER 23

EXERCISE 23-1 (10–15 minutes)

(a) Operating—add to net income.

(b) Financing activity.

(c) Investing activity.

(d) Operating—add to net income.

(e) Significant noncash investing and financing activity.

(f) Financing activity.

(g) Operating—add to net income.

(h) Financing activity.

(i) Significant noncash investing and financing activity.

(j) Financing activity.

(k) Operating—deduct from net income.

(l) Investing activity.

EXERCISE 23-5 (20–30 minutes)

NORMAN COMPANY

Partial Statement of Cash Flows

For the Year Ended December 31, 2010

Cash flows from operating activities

Cash receipts from customers .............................. $862,000 (a)

Cash payments ......................................................

For operating expenses ............................... $609,000 (b)

For income taxes.......................................... 44,500 (c) 653,500

Net cash provided by operating

activities ............................................................. $208,500

Page 61: Solution manual for intermediate accounting-14e-kieso-warfield-weygandt

58

(a) Computation of cash receipts from customers:

Revenue from fees .................................................................... $840,000

Add: Decrease in accounts receivable

Ad ($59,000 – $37,000) ..................................................... 22,000

Cash receipts from customers ................................................... $862,000

(b) Computation of cash payments:

Operating expenses per income statement .............................. $624,000

Deduct: Increase in accounts payable

Deduct ($46,000 – $31,000) .............................................. 15,000

Cash payments for operating expenses .................................... $609,000

(c) Income tax expense per income statement .............................. $ 40,000

Add: Decrease in income taxes payable

Add ($8,500 – $4,000) ......................................................... 4,500

Cash payments for income taxes .............................................. $ 44,500

EXERCISE 23-6 (15–20 minutes)

NORMAN COMPANY

Partial Statement of Cash Flows

For the Year Ended December 31, 2010

Cash flows from operating activities

Net income ....................................................................................... $90,000

Adjustments to reconcile net income

to net cash provided by operating activities:

Depreciation expense ................................................................. $60,000

Loss on sale of equipment .......................................................... 26,000

Decrease in accounts receivable ................................................. 22,000

Increase in accounts payable ...................................................... 15,000

Decrease in income taxes payable .............................................. (4,500) 118,500

Net cash provided by operating activities ........................................ $208,500

EXERCISE 23-11 (30–35 minutes)

FAIRCHILD COMPANY

Statement of Cash Flows

For the Year Ended December 31, 2010

Page 62: Solution manual for intermediate accounting-14e-kieso-warfield-weygandt

59

(Indirect Method)

Cash flows from operating activities

Net income .................................................................................................. $ 810

Adjustments to reconcile net income to net cash

provided by operating activities:

Depreciation expense ($1,200 – $1,170) ............................................. $ 30

Gain on sale of investments ................................................................. (80)

Decrease in inventory ........................................................................... 300

Increase in accounts payable ............................................................... 400

Increase in receivables ......................................................................... (450)

Decrease in accrued liabilities .............................................................. (50) 150

Net cash provided by operating activities ................................................... 960

Cash flows from investing activities

Sale of held-to-maturity investments

[($1,470 – $1,300) + $80] .........................................................................

250

Purchase of plant assets [($1,900 – $1,700) – $70] .................................... (130)

Net cash provided by investing activities .................................................... 120

Cash flows from financing activities

Issuance of capital stock [($1,900 – $1,700) – $70] ...................................... 130

Retirement of bonds payable ........................................................................ (250)

Payment of cash dividends ............................................................................ (260)

Net cash used by financing activities ............................................................ (380)

Net increase in cash................................................................................................... 700

Cash, January 1, 2010 ................................................................................................ 1,100

Cash, December 31, 2010 .......................................................................................... $1,800

Noncash investing and financing activities

Issuance of common stock for plant assets .................................................. $ 70


Top Related